Đến nội dung

Hình ảnh

Inequalities From 2016 Mathematical Olympiads

* * * * * 3 Bình chọn

  • Please log in to reply
Chủ đề này có 139 trả lời

#41
Nguyenhuyen_AG

Nguyenhuyen_AG

    Trung úy

  • Thành viên nổi bật 2016
  • 945 Bài viết

 Bài này quen thuộc

 

Bất đẳng thức này đúng là quen thuộc, có thể tham khảo thêm đề Việt Nam MO 1998.

 

Bài 19 (Cyprus TST). Giả sử $a,\,b,\,c$ là độ dài ba cạnh của tam giác $ABC$ và thỏa mãn điều kiện
$$a\sqrt{8}+b\sqrt{6}+c\sqrt{2}\ge 4\sqrt{a^2+b^2+c^2},$$
Chứng minh rằng $ABC$ là tam giác vuông.

 


Nguyen Van Huyen
Ho Chi Minh City University Of Transport

#42
dark templar

dark templar

    Kael-Invoker

  • Hiệp sỹ
  • 3788 Bài viết

Bất đẳng thức này đúng là quen thuộc, có thể tham khảo thêm đề Việt Nam MO 1998.

 

Bài 19 (Cyprus TST). Giả sử $a,\,b,\,c$ là độ dài ba cạnh của tam giác $ABC$ và thỏa mãn điều kiện
$$a\sqrt{8}+b\sqrt{6}+c\sqrt{2}\ge 4\sqrt{a^2+b^2+c^2},$$
Chứng minh rằng $ABC$ là tam giác vuông.

 

 

Theo C-S:

$$4\sqrt{a^{2}+b^{2}+c^{2}}=\sqrt{8+6+2}\sqrt{a^{2}+b^{2}+c^{2}}\geqslant a\sqrt{8}+b\sqrt{6}+c\sqrt{2}$$

 

Kết hợp với điều kiện đầu bài cho,ta sẽ có đẳng thức xảy ra hay:

$$\frac{a}{\sqrt{8}}=\frac{b}{\sqrt{6}}=\frac{c}{\sqrt{2}}\Leftrightarrow \frac{a^{2}}{8}=\frac{b^{2}}{6}=\frac{c^{2}}{2}=\frac{b^{2}+c^{2}}{8}\Rightarrow a^{2}=b^{2}+c^{2}$$

 

Vậy tam giác ABC vuông.

 

Bài 15 (Australien MO). Cho $a,b$ là hai số thực thỏa mãn $a^{2}+b^{2}=1.$ Chứng minh rằng

$$\left | a+\frac{a}{b}+b+\frac{b}{a} \right |\geq 2-\sqrt{2}.$$

 

Theo BĐT trị tuyệt đối và C-S:

$$\left | (a+b)+\left ( \frac{a}{b}+\frac{b}{a} \right ) \right |\geqslant \left | \frac{a}{b}+\frac{b}{a} \right |-\left | a+b \right |\geqslant 2-\sqrt{2(a^{2}+b^{2})}=2-\sqrt{2}$$


"Do you still... believe in me ?" Sarah Kerrigan asked Jim Raynor - Starcraft II:Heart Of The Swarm.

#43
minhrongcon2000

minhrongcon2000

    Thượng sĩ

  • Thành viên
  • 213 Bài viết

Bài 3 (Korea Winter Program Practice Test). Cho ba số thực không âm $x,\,y,\,z$ thỏa mãn

\[(x+y-1)^2+(y+z-1)^2+(z+x-1)^2=27.\]
Tìm giá trị lớn nhất và giá trị nhỏ nhất của biểu thức $x^4+y^4+z^4.$

Giải:

 

$(x+y-1)^{2}+(y+z-1)^{2}+(z+x-1)^{2}=27 \Leftrightarrow x^{2}+y^{2}+z^{2}+(x+y+z)^{2}-4(x+y+z)=24$

 

Áp dụng bất đẳng thức B.C.S, ta có:

 

$24\geqslant \frac{(x+y+z)^{2}}{3}+(x+y+z)^{2}-4(x+y+z)\Leftrightarrow x+y+z\leqslant 6$

 

Do đó, $x^{2}+y^{2}+z^{2}=24-(x+y+z)^{2}+4(x+y+z) \leqslant 12$

 

Chú ý rằng, ta cũng có hai bất đẳng thức là $x+y+z \geqslant \sqrt{x^{2}+y^{2}+z^{2}}$ và $x+y+z+\sqrt{x^{2}+y^{2}+z^{2}}\geqslant 2\sqrt{x^{2}+y^{2}+z^{2}}>4$. Do đó,

 

$(x+y+z-\sqrt{x^{2}+y^{2}+z^{2}})(x+y+z+\sqrt{x^{2}+y^{2}+z^{2}}-4)\geqslant 0$

 

$\Leftrightarrow (x+y+z)^{2}-4(x+y+z)\geqslant x^{2}+y^{2}+z^{2}-4\sqrt{x^{2}+y^{2}+z^{2}}$

$\Leftrightarrow 24-(x^{2}+y^{2}+z^{2})\geqslant x^{2}+y^{2}+z^{2}-4\sqrt{x^{2}+y^{2}+z^{2}}$

$\Leftrightarrow x^{2}+y^{2}+z^{2}\leqslant (\sqrt{13}+1)^{2}$

 

Ta lại có được $\frac{(x^{2}+y^{2}+z^{2})^{2}}{3} \leqslant x^{4}+y^{4}+z^{4} \leqslant (x^{2}+y^{2}+z^{2})^{2}$

 

Do đó, ta có được giá trị lớn nhất của $x^{4}+y^{4}+z^{4}$ là $(\sqrt{13}+1)^{4}$ khi $x=y=0,z=\sqrt{13}+1$ và các hoán vị

Giá trị nhỏ nhất của $x^{4}+y^{4}+z^{4}$ là $48$ khi $x=y=z=2$


Bài viết đã được chỉnh sửa nội dung bởi minhrongcon2000: 10-04-2016 - 22:11

$\lim_{x \to \infty } Love =+\infty$


#44
IMOer

IMOer

    Hạ sĩ

  • Thành viên
  • 50 Bài viết

 

Bài 7 (Hong Kong TST). Với $n$ là một số nguyên dương, giả sử tồn tại $n$ số thực dương $x_1,\,x_2,\,\ldots,\,x_n$ khác nhau sao cho mọi số nguyên $1 \leqslant i,\,j \leqslant n,$ thì $$(3x_i-x_j) (x_i-3x_j)\geqslant (1-x_ix_j)^2.$$ Tìm giá trị lớn nhất có thể có của $n.$

 

Không mất tính tổng quát, giả sử ${{x}_{1}}<{{x}_{2}}<...<{{x}_{n}}$.

Với mọi $n\ge i>j>1$ ta có:

$\left( 3{{x}_{i}}-{{x}_{j}} \right)\left( {{x}_{i}}-3{{x}_{j}} \right)\ge {{\left( 1-{{x}_{i}}{{x}_{j}} \right)}^{2}}\Leftrightarrow 3{{\left( {{x}_{i}}-{{x}_{j}} \right)}^{2}}\ge {{\left( 1+{{x}_{i}}{{x}_{j}} \right)}^{2}}\Leftrightarrow \frac{{{x}_{i}}-{{x}_{j}}}{1+{{x}_{i}}{{x}_{j}}}\ge \frac{1}{\sqrt{3}}$

Vì ${{x}_{i}}>0$ nên tồn tại ${{\alpha }_{i}}\in \left( 0;\frac{\pi }{2} \right)$ sao cho ${{x}_{i}}=\tan {{\alpha }_{i}}$.

Khi đó: $\tan \left( {{\alpha }_{i}}-{{\alpha }_{j}} \right)\ge \frac{1}{\sqrt{3}}\Rightarrow {{\alpha }_{i}}-{{\alpha }_{j}}\ge \frac{\pi }{6}$ với mọi $i>j$.

Nếu $n\ge 4$ thì ${{\alpha }_{4}}-{{\alpha }_{1}}=\left( {{\alpha }_{4}}-{{\alpha }_{3}} \right)+\left( {{\alpha }_{3}}-{{\alpha }_{2}} \right)+\left( {{\alpha }_{2}}-{{\alpha }_{1}} \right)\ge \frac{\pi }{2}\Rightarrow {{\alpha }_{4}}> \frac{\pi }{2}$, vô lý.

Vậy suy ra: $n\le 3$.

Với $n=3$, ta có thể chọn: ${{x}_{1}}=\tan \frac{\pi }{12};\ {{x}_{2}}=\tan \frac{\pi }{4};\ {{x}_{3}}=\tan \frac{5\pi }{12}$ thỏa mãn bài toán.



#45
IMOer

IMOer

    Hạ sĩ

  • Thành viên
  • 50 Bài viết

Bài 1 (Japan MO Preliminary). Với $a,\,b,\,c,\,d$ là bốn số thực thỏa mãn

\[\left\{\begin{aligned}&(a+b)(c+d)=2\\&(a+c)(b+d)=3\\&(a+d)(b+c)=4\end{aligned}\right.\]
Tìm giá trị nhỏ nhất của biểu thức $a^2+b^2+c^2+d^2.$

 

Từ giả thiết đã cho ta suy ra: $2\sum{ab}=9$

Và $4=\left( a+d \right)\left( b+c \right)\le \frac{{{\left( a+b+c+d \right)}^{2}}}{4}\Rightarrow {{\left( a+b+c+d \right)}^{2}}\ge 16$

Lại có: $\sum{{{a}^{2}}}={{\left( \sum{a} \right)}^{2}}-2\sum{ab}\ge 16-9=7$

Dấu bằng xảy ra chẳng hạn khi: $a=\frac{1-\sqrt{2}}{2};\ b=\frac{3-\sqrt{2}}{2};\ c=\frac{1+\sqrt{2}}{2};\ d=\frac{3+\sqrt{2}}{2}$ 



#46
Nguyenhuyen_AG

Nguyenhuyen_AG

    Trung úy

  • Thành viên nổi bật 2016
  • 945 Bài viết

Bài 20 (China Mathematical Olympiad). Cho $a_1,a_2,\ldots, a_{31},\,b_1,b_2, \ldots, b_{31}$ là các số nguyên dương thỏa mãn
$$a_1< a_2<\cdots< a_{31}\leqslant 2015,\; b_1< b_2<\cdots<b_{31} \leqslant 2015,$$

$$a_1+a_2+\cdots+a_{31}=b_1+b_2+\cdots+b_{31}.$$
Tìm giá trị lớn nhất của $S=|a_1-b_1|+|a_2-b_2|+\cdots+|a_{31}-b_{31}|.$


Nguyen Van Huyen
Ho Chi Minh City University Of Transport

#47
IMOer

IMOer

    Hạ sĩ

  • Thành viên
  • 50 Bài viết

Bài 21 (European Girls' Mathematical Olympiad): Cho $n$ là số nguyên dương lẻ và $x_1,x_2,...,x_n$ là các số thực không âm.

Chứng minh rằng: $\min_{1\le i\le n} (x_i^2+x_{i+1}^2)\le\max_{1\le j\le n} (2x_jx_{j+1})$



#48
Nguyenhuyen_AG

Nguyenhuyen_AG

    Trung úy

  • Thành viên nổi bật 2016
  • 945 Bài viết

Chúng ta có nên thêm bất đẳng thức hình học vào luôn không các bạn ?


Nguyen Van Huyen
Ho Chi Minh City University Of Transport

#49
trananhduong62

trananhduong62

    Hạ sĩ

  • Thành viên
  • 81 Bài viết

Bài 1

Hình gửi kèm

  • 14.jpg

trananhduong62 :icon6:  :icon6:  :icon6:  :ukliam2: GOOD!


#50
trananhduong62

trananhduong62

    Hạ sĩ

  • Thành viên
  • 81 Bài viết

Bài 15

Hình gửi kèm

  • 17.jpg

trananhduong62 :icon6:  :icon6:  :icon6:  :ukliam2: GOOD!


#51
Nguyenhuyen_AG

Nguyenhuyen_AG

    Trung úy

  • Thành viên nổi bật 2016
  • 945 Bài viết

Bài 22 (China Zhejiang High School Mathematics Competition). Cho ba số nguyên $a, b, c $ khác nhau. Tìm giá trị nhỏ nhất của $4(a^2+b^2+c^2)-(a+b+c)^2.$
 


Nguyen Van Huyen
Ho Chi Minh City University Of Transport

#52
IMOer

IMOer

    Hạ sĩ

  • Thành viên
  • 50 Bài viết

Bài 22 (China Zhejiang High School Mathematics Competition). Cho ba số nguyên $a, b, c $ khác nhau. Tìm giá trị nhỏ nhất của $4(a^2+b^2+c^2)-(a+b+c)^2.$
 

Đặt: $S=4(a^2+b^2+c^2)-(a+b+c)^2$.

Ta có: $S=(a-b)^2+(b-c)^2+(c-a)^2+a^2+b^2+c^2$.

Vì $a,b,c$ là các số nguyên khác nhau nên $|a-b|\ge 1; |b-c|\ge 1; |a-c|\ge 1$.

Và dấu bằng không thể đồng thời xảy ra ở 3 bất đằng thức trên nên trong 3 số $|a-b|; |b-c|; |c-a|$ có ít nhất một số lớn hơn hoặc bằng 2.

Suy ra: $(a-b)^2+(b-c)^2+(c-a)^2\ge 1^2+1^2+2^2=6$.

Đồng thời, $a,b,c$ là các số nguyên phân biệt nên: $a^2+b^2+c^2\ge 0^2+1^2+(-1)^2=2$.

Suy ra: $S\ge 8$.

Dấu bằng xảy ra khi và chỉ khi: $\{a, b, c\}=\{1, 0, -1\}$.



#53
Nguyenhuyen_AG

Nguyenhuyen_AG

    Trung úy

  • Thành viên nổi bật 2016
  • 945 Bài viết

Bài 23 (Romania JBMO TST 2016). Với $m,n$ là hai số tự nhiên và ba số thực $x,y,z$ thuộc $[0,1].$ Chứng minh rằng
\[0 \leqslant x^{m+n}+y^{m+n}+z^{m+n}-x^my^n-y^mz^n-z^mx^n \leqslant 1.\]
Đẳng thức xảy ra khi nào ?

 

Bài 24 (China Junior High School Mathematics League). Cho ba số thực $x,y$ và $z$ thỏa mãn điều kiện $x+y+z=1.$ Tìm giá trị lớn nhất của biểu thức $xy+2yz+3zx.$
 


Bài viết đã được chỉnh sửa nội dung bởi Nguyenhuyen_AG: 24-04-2016 - 19:16

Nguyen Van Huyen
Ho Chi Minh City University Of Transport

#54
canhhoang30011999

canhhoang30011999

    Thiếu úy

  • Thành viên
  • 634 Bài viết

Bài 23 (Romania JBMO TST 2016). Với $m,n$ là hai số tự nhiên và ba số thực $x,y,z$ thuộc $[0,1].$ Chứng minh rằng
\[0 \leqslant x^{m+n}+y^{m+n}+z^{m+n}-x^my^n-y^mz^n-z^mx^n \leqslant 1.\]
Đẳng thức xảy ra khi nào ?

 

Bài 24 (China Junior High School Mathematics League). Cho ba số thực $x,y$ và $z$ thỏa mãn điều kiện $x+y+z=1.$ Tìm giá trị lớn nhất của biểu thức $xy+2yz+3zx.$
 

24 ta chứng minh $xy+2yz+3zx \leq \frac{3} {4}$

tức là chứng minh $(x+y+z)^{2}\geq \frac{4}{3}(xy+2yz+3zx)$

hay $x^{2} +x(\frac{2}{3}y-2z)+y^{2}+z^{2}-\frac{2}{3}yz \geq 0$ với mọi $x$ thuộc R

tức là $\bigtriangleup \leq 0$

hay $\frac{-32}{9}y^{2} \leq 0$(luôn đúng)

dấu bằng xảy ra khi $x=z=\frac{1}{2},y=0$



#55
Nguyenhuyen_AG

Nguyenhuyen_AG

    Trung úy

  • Thành viên nổi bật 2016
  • 945 Bài viết

Bài 25 (Croatia TST). Cho số nguyên $n \geqslant 1$ và $x_1,x_2, \ldots, x_n$ là các số thực không âm. Chứng minh rằng \[\left(x_1 + \frac{x_2}{2} + \cdots + \frac{x_n}{n}\right)\left(x_1 + 2x_2 + \cdots + nx_n\right) \leqslant \frac{(n+1)^2}{4n} (x_1 + x_2 + \cdots + x_n)^2.\]
 


Nguyen Van Huyen
Ho Chi Minh City University Of Transport

#56
IMOer

IMOer

    Hạ sĩ

  • Thành viên
  • 50 Bài viết

Bài 26 (Azerbaijan JMO). Cho $n\in\mathbb{N}$. Chứng minh rằng:

\[n\sqrt[n+1]{n+2}+\sqrt[n+1]{n+2}-1<n+1+\dfrac{1}{2}+\dfrac{1}{3}+\cdots+\dfrac{1}{n+1}\]



#57
Nguyenhuyen_AG

Nguyenhuyen_AG

    Trung úy

  • Thành viên nổi bật 2016
  • 945 Bài viết

Bài 27 (Iran Second Round). Cho ba số thực $0<a \leqslant b \leqslant c.$ Chứng minh rằng
$$\frac{(c-a)^2}{6c}\leq \frac{a+b+c}{3}-\frac{3}{\frac{1}{a}+\frac{1}{b}+\frac{1}{c}}.$$

 


Nguyen Van Huyen
Ho Chi Minh City University Of Transport

#58
revenge

revenge

    Hạ sĩ

  • Thành viên
  • 76 Bài viết

Bài 26 (Azerbaijan JMO). Cho $n\in\mathbb{N}$. Chứng minh rằng:

\[n\sqrt[n+1]{n+2}+\sqrt[n+1]{n+2}-1<n+1+\dfrac{1}{2}+\dfrac{1}{3}+\cdots+\dfrac{1}{n+1}\]

bất đẳng thức cần chứng minh tương đương

$2+\frac{3}{2}+\frac{4}{3}+...+\frac{n+2}{n+1}>(n+1)\sqrt[n+1]{n+2}$

đúng theo AM-GM nhưng đấu bằng không xảy ra



#59
Nguyenhuyen_AG

Nguyenhuyen_AG

    Trung úy

  • Thành viên nổi bật 2016
  • 945 Bài viết

Bài 28 (Russia). Cho bốn số thực dương $a, b, c, d$ thỏa mãn điều kiện $a+b+c+d=3.$ Chứng minh rằng $$\frac{1}{a^2}+\frac{1}{b^2}+\frac{1}{c^2}+\frac{1}{d^2}\le\frac{1}{a^2b^2c^2d^2},$$


$$\frac{1}{a^3}+\frac{1}{b^3}+\frac{1}{c^3}+\frac{1}{d^3}\le\frac{1}{a^3b^3c^3d^3}.$$


Nguyen Van Huyen
Ho Chi Minh City University Of Transport

#60
canhhoang30011999

canhhoang30011999

    Thiếu úy

  • Thành viên
  • 634 Bài viết

Bài 25 (Croatia TST). Cho số nguyên $n \geqslant 1$ và $x_1,x_2, \ldots, x_n$ là các số thực không âm. Chứng minh rằng \[\left(x_1 + \frac{x_2}{2} + \cdots + \frac{x_n}{n}\right)\left(x_1 + 2x_2 + \cdots + nx_n\right) \leqslant \frac{(n+1)^2}{4n} (x_1 + x_2 + \cdots + x_n)^2.\]
 

ta có $(x_{1}+2x_{2}+...+nx_{n})(nx_{1}+\frac{n}{2}x_{2}+...+x_{n})\leq \frac{((n+1)x_{1}+(\frac{n}{2}+2)x_{2}+...+(n+1)x_{n})^{2}}{4}$

$\frac{\leq (n+1)^{2}(x_{1}+x_{2}+...+x_{n})^{2}}{4}$ (dpcm)

dấu bằng xảy ra khi $x_1=x_n=1,x_{2}=x_{3}=...=x_{n-1}=0$ 






1 người đang xem chủ đề

0 thành viên, 1 khách, 0 thành viên ẩn danh